Vi sono chiaramente alcuni collegamenti tra la continuità destra e sinistra e la continuità classica,
così come ve ne sono tra il limite destro e sinistro ed il limite classico. \\
\li$\xbar$ punto di accumulazione destro o sinistro di $X$$\iff$$\xbar$ punto di accumulazione di $X$, \\
\li$\xbar$ punto di accumulazione destro e sinistro di $X$$\implies$$\xbar$ punto di accumulazione di $X$ (non
è però per forza vero il contrario, è sufficiente considerare $0$ in $(0, \infty)$), \\
è però per forza vero il contrario, è sufficiente considerare $(0, \infty)$, dove $0$ è solo un punto di
accumulazione destro), \\
\li$f$ è continua in $\xbar$$\iff$$f$ è continua sinistra e destra in $\xbar$, \\
\li se $\xbar$ è un punto di accumulazione destro e sinistro, $\lim_{x \to\xbar} f(x)= L \iff\lim_{x \to\xbar^+} f(x)= L$ e $\lim_{x \to\xbar^-} f(x)= L$, \\
\li se $\xbar$ è un punto di accumulazione solo destro, $\lim_{x \to\xbar} f(x)= L \iff\lim_{x \to\xbar^+} f(x)= L$, \\
@ -354,17 +356,36 @@
\vskip 0.05in
ossia la funzione indicatrice dell'insieme $\QQ$ in $\RR$.
%TODO: aggiungere dimostrazione.
ossia la funzione indicatrice dell'insieme $\QQ$ in $\RR$. Si dimostra che $f$ non è continua
in nessun punto di $\RR$. Sia infatti $\xbar\in\RR\setminus\QQ$. Dal momento che $\QQ$ è denso
in $\RR$, $\xbar$ è un punto di accumulazione di $\QQ$, e quindi esiste una successione $(x_n)\subseteq\QQ$
tale che $x_n \tendston\xbar$. Se $f$ fosse continua in $\xbar$, $\lim_{n \to\infty} f(x_n)=0$,
ma per l'intorno $I =[-\frac{1}{2}, \frac{1}{2}]$ non esiste alcun $n_k$ tale per cui $f(x_n)\in I$$\forall n
\geq n_k$, dal momento che, per definizione di $f$, $f(x_n) = 1$$\forall n \in\NN$. Quindi $f$ non è continua
in nessun $\xbar\in\RR\setminus\QQ$. \\
Sia ora $\xbar\in\QQ$. $\xbar$ è un punto di accumulazione di $\RR\setminus\QQ$ (si può infatti
considerare la successione $(x_n)\subseteq\RR\setminus\QQ$ definita da $x_n =\xbar+\frac{\sqrt{2}}{n}$,
che è tale che $x_n \tendston\xbar$). Analogamente a come visto prima, allora, per l'intorno $I =[\frac{1}{2}, \frac{3}{2}]$, $f(x_n)\notin I$$\forall n \in\NN$, e quindi $f$ non è continua neanche su $\xbar\in\QQ$.
\end{example}
\begin{exercise}
Mostrare che l'insieme dei punti di discontinuità di una funzione $f : X \to\RR$ monotona è al più
numerabile.
Mostrare che l'insieme dei punti di discontinuità di una funzione $f : I\to\RR$ monotona è al più
numerabile, dove $I$ è un intervallo.
\end{exercise}
%TODO: aggiungere risoluzione
\begin{solution}
Si assuma $f$ crescente, senza perdita di generalità (altrimenti è sufficiente considerare $g(x)=-f(x)$).
Sia $E$ l'insieme dei punti di discontinuità di $f$. $\forall\xbar\in E$, $\xbar$ è un punto di accumulazione
destro e sinistro di $I$ (infatti $I$ è un intervallo), ed in particolare esistono sempre il limite destro $L^-(\xbar)$
ed il limite sinistro $L^+(\xbar)$ in $\xbar$ (dal momento che $f$ è monotona), e sono tali che $L^+(\xbar) > L^-(\xbar)$ (sicuramente
sono diversi, altrimenti
$f$ sarebbe continua in $\xbar$; inoltre $f$ è crescente). Allora sia $f : E \to\QQ$ tale che $\xbar\mapsto c$, dove $c \in\QQ$ è
un punto razionale in $(L^-(\xbar), L^+(\xbar))$ (tale $c$ esiste sempre, per la densità di $\QQ$ in $\RR$).
Inoltre, $x < y \implies L^+(x)\leq L^-(y)$, e quindi ogni intervallo da cui è preso $c$ è distinto al variare
di $\xbar\in E$. Quindi $f$ è iniettiva, e vale $\abs{E}\leq\abs{\QQ}=\abs{\NN}$. Si conclude allora
che $E$ è al più numerabile.
\end{solution}
\begin{theorem} (della permanenza del segno)
Data $(x_n)\subseteq\RR$ tale che $x_n \tendston L > 0$, allora
@ -414,7 +435,21 @@
la tesi.
\end{proof}
%TODO: aggiungere dimostrazione alternativa con il metodo della bisezione.
\begin{proof}[Dimostrazione alternativa] (metodo di bisezione per la ricerca degli zeri)
Come prima, senza alcuna perdita di generalità, si pone $f(a) < 0 < f(b)$. Si ponga $x_0=\frac{a+b}{2}$,
$I_0=(a, b)$.
Se $f(x_0)=0$, allora il teorema è dimostrato. Altrimenti, $f(x_0) > 0$ o $f(x_0) < 0$. Nel primo caso,
si consideri $I_1=(a, x_0)$, altrimenti si ponga $I_1=(x_0, b)$. Si riapplichi allora l'algoritmo con $a :=\inf I_1$ e $b :=\sup I_1$, definendo la successione $(x_n)$ e gli intervalli $I_n$ per ogni passo $n$ dell'algoritmo. \\
Se la successione $(x_n)$ è finita, allora $\exists n \mid f(x_n)=0$, e quindi il teorema è dimostrato. Altrimenti,
si osservi che la successione degli intervalli è decrescente, e che $\abs{I_n}=\frac{b-a}{2^n}\tendston0$:
allora, poiché $x_n \in I_n$$\forall n \in\NN$, $(x_n)$ ammette limite. In particolare, $I_n \tendston\{c\}$,
e quindi $x_n \tendston c \in I_0$. Siano $a_n$, $b_n$ le successioni tali che $I_n =(a_n, b_n)$$\forall n \in\NN$.
Vale in particolare che $a_n, b_n \tendston c$. Allora, per la continuità di $f$ su $(a, b)$, vale che
$\lim_{n \to\infty} f(a_n)= f(c)$ e che $\lim_{n \to\infty} f(b_n)= f(c)$: poiché ogni elemento di $(a_n)$ è
per costruzione tale che $f(a_n) < 0$, deve valere che $f(x)\leq0$ per il teorema della permanenza del segno; analogamente deve valere per costruzione di $(b_n)$ che $f(c)\geq0$. Si conclude allora che $f(c)=0$, da cui
la tesi.
\end{proof}
\begin{corollary} (dei valori intermedi) Dati $I =(a, b)$ e
$f : I \to\RRbar$ continua, allora $y_1$, $y_2\in f(I)\implies